Find angle A to the nearest tenth.
(Show work if you can plss)

Find Angle A To The Nearest Tenth. (Show Work If You Can Plss)

Answers

Answer 1

Answer:

∠ A ≈ 36.9°

Step-by-step explanation:

assuming the triangle to be right at ∠ C

using the sine ratio in the right triangle

sinA = \(\frac{opposite}{hypotenuse}\) = \(\frac{BC}{AB}\) = \(\frac{6}{10}\) , then

∠ A = \(sin^{-1}\) ( \(\frac{6}{10}\) ) ≈ 36.9° ( to the nearest tenth )


Related Questions

consider the function f(x) =3x+8, What is the correct set up for f(x)=7?

Answers

Answer: 29

Step-by-step explanation: 3 times 7 = 21 + 8 = 29

Answer:

7=3x+8

x=-1/3

Step-by-step explanation:

we just replace f(X) with 7

7=3x+8

-8.    -8

-1=3x

/3. /3

-1/3=x

Hopes this helps please mark brainliest

ronnie and tanya both identically priced cans of chili and identically priced jars of salsa to make a dip. ronnie bought 3 cans of chili and 2 jars of salsa for $15.12. tanya bought 2 cans of chili and 4 jars of salsa for $19.36. write a system of equations that could be used to find, x, the cost of one can of chili, and y, the cost of one jar of salsa. responses

Answers

The system of linear equations that can be formed to find x and y is 3x + 2y = 15.12 and 2x + 4y = 19.36. Then one can of chili costs $2.37 and one jar of salsa costs $4.005.

Let us consider  x being  the cost of one can of chili and y be the cost of one jar of salsa.
Then we can write two equations
3x + 2y = 15.12
2x + 4y = 19.36

We can multiply the first linear equation by 2 and subtract it from the second  linear equation in order to  eliminate x
(2x + 4y = 19.36) - 2(3x + 2y = 15.12)
= -4x + 0y
= -9.48

Applying Simplification to this equation
-4x = -9.48
Dividing both sides by -4 gives us:
x = 2.37

Now we can place this value of x into either of the original equations to find y
3(2.37) + 2y = 15.12
7.11 + 2y = 15.12
Subtracting 7.11 from both sides
2y = 8.01
Dividing both sides by 2
y = 4.005
Then, one can of chili costs $2.37 and one jar of salsa costs $4.005.


To learn more about linear equation,
https://brainly.com/question/28732353
#SPJ4

whats the mean of the numbers 3 7 2 4 7 5 7 1 8 8

Answers

Answer:

5.2

Step-by-step explanation:

adding all the numbers together and dividing it by 10.

Answer:

mean = 5.2

Step-by-step explanation:

The mean (or average) of a group of numbers is defined as the value calculated by adding all the given numbers together and then dividing the result by the number of numbers given.

Therefore,

\(\boxed{\mathrm{mean = \frac{sum \ of \ the \ numbers}{number \ of \ numbers}}}\).

In the question, the numbers given are: 3, 7, 2, 4, 7, 5, 7, 1, 8, and 8.

Therefore,

sum = 3 + 7 + 2 + 4 + 7 + 5 + 7 + 1 + 8 + 8

       = 52

There are 10 numbers given in the question. Therefore, using the formula given above, we can calculate the mean:

\(\mathrm{mean = \frac{52}{10}}\)

            \(= \bf 5.2\)

Hence, the mean of the given numbers is 5.2.

HELP SOLVE FOR THE VALUE OF X

HELP SOLVE FOR THE VALUE OF X

Answers

68 degrees. They’re the same

PLEASE HELP ASAP!!!!!!! 26 points

If Serena puts $1000 in a savings account that pays 2% interest compounded annualy, how much money will she have in the account after 10 years? (Hint: To find the amount of money after 1 year. multiply by 1.02.)

Answers

Answer:

5000

Step-by-step explanation:

1000 ÷ 2 = 500 x 10 = 5000

Answer:

$1221.19

Step-by-step explanation:

Formula for compound interest:

A= P(1+r/n)^n x t

A = final amount

P = initial principal balance

r = interest rate

n = number of times interest applied per time period

t = number of time periods elapsed

We know that Serena pays annually meaning yearly (12 months or 1 year)

So after substituting all the numbers in,

A= 1000(1+0.02/12)^(12x10)

We change the rate to 0.02 because we must divide 2% by 100%. And divide the rate by number of times applied per time (n).

DISCLAIMER:

I have not done compound interest questions in a long time but if you want to confirm my answer, go to a calculator site and just input the numbers on there.

An electrician charges a set fee of $150 for every house call and then charges an additional $70 per hour. Write a function for C, in terms of t, representing the total cost of the electrician's services if the electrician spends t hours at the house working. How much would a 4 hour job cost?

Answers

The function C(t) that models the the total cost of the electrician's services for spending [t] hours is C(t) = 70t + 150 and the total cost of four hour work will be $430.

What is Equation Modelling?

Equation modelling is the process of writing a mathematical verbal expression in the form of a mathematical expression for correct analysis, observations and results of the given problem.

Given is an electrician who charges a set fee of $150 for every house call and then charges an additional $70 per hour.

Assume that function C(t) represents the function that models the the total cost of the electrician's services for spending [t] hours at house working.

Fixed rate of electrician = $150

Rate per hour = $70

Now, we can write the function C(t) as -

C(t) = 70t + 150

For a four hour job, we have t = 4. We can write -

C(4) = 70 x 4 + 150

C(4) = 280 + 150

C(4) = 430

For 4 hour work, the electrician will cost $430.

Therefore, the function C(t) that models the the total cost of the electrician's services for spending [t] hours is C(t) = 70t + 150 and the total cost of four hour work will be $430.

To solve more questions on equation modelling , visit the link below-

https://brainly.com/question/18057354

#SPJ1

For the function f(x) = 3x^2 - 7, find the value of f(x) when x = 3. A 20 B 27 C 3 D 3/7

Answers

Explanation

\(f(x)=3x^2-7\)

Step 1

to solve this, we need replace the x value, with the given value,so

Solve: 9m - 3 (m + 6) = 2 (m + 7)

Answers

X is m in your case but the answer is 8 (:
Solve: 9m - 3 (m + 6) = 2 (m + 7)

Answer:

\(m=8\)

Step-by-step explanation:

\(9m-3(m+6)=2(m+7)\)

\(9m-3m-18=2m+14\)

\(6m-18=2m+14\)

\(4m= 32\)

\(m=8\)

According to the graph, what is the constant of proportionality?
(0,0) (5,3)

According to the graph, what is the constant of proportionality?(0,0) (5,3)

Answers

Answer:

3/5

Step-by-step explanation:

The constant of proportionality is the slope

m = (y2-y1)/(x2-x1)

   = (3-0)/(5-0)

   =3/5

AB bearing of N45°35'E bearing BC has a of 5 8 1° 36' E What is the angle formed by these bearings? What is Azimuthas of thes bearings?

Answers

The angle formed by the bearings AB and BC is N28°59'E. The azimuth of the bearings is N58°34'E.

To find the angle formed by the bearings, we subtract the bearing of BC from the bearing of AB.

The bearing of AB is N45°35'E, and the bearing of BC is 5°36'E.

To subtract these angles, we convert them to decimal degrees.

N45°35'E is equal to 45.58°, and 5°36'E is equal to 5.60°.

Now we subtract 5.60° from 45.58° to find the angle formed by the bearings.

45.58° - 5.60° = 39.98°

So, the angle formed by the bearings AB and BC is approximately 39.98°.

To find the azimuth of the bearings, we take the average of the two bearings.

N45°35'E is equal to 45.58°, and 5°36'E is equal to 5.60°.

Adding these two angles and dividing by 2 gives us the azimuth.

(45.58° + 5.60°) / 2 = 51.18°

Therefore, the azimuth of the bearings is N58°34'E.

Know more about angle here:

https://brainly.com/question/31818999

#SPJ11

Divide. Reduce the answer to lowest terms. 4/5 divided by 16/15

Answers

Answer:

Your answer is 3/4  or  0.75

Step-by-step explanation:

4/5  /   16/15

4/5  *   15/16

Cross simplify.

What is the image of (-8,2)(−8,2) after a reflection over the y-axis?

Answers

Answer:

(8,2)

Step-by-step explanation:

The rule is (x,y) becomes (-x, y).  You can visualize this.  (-8,2) is 8 units to the left of the origin (0,0) and two units up from the origin (0,0).  Now, it we flip it so that is is on the other side of the y axis, it is still two units above the origin(0,0), but it is now 8 units to the right of the origin.

given p : q = 5 : 2 and q : r = 4 : 3, find the ratio of p : q : r​

Answers

Answer:

p : q : r = 10 : 4 : 3

Step-by-step explanation:

Expressing the ratio in fractional form

\(\frac{q}{r}\) = \(\frac{4}{3}\) ( cross- multiply )

4r = 3q ( divide both sides by 4 )

r = \(\frac{3}{4}\) q = \(\frac{3}{4}\) × 2 = \(\frac{3}{2}\)

Then

p : q : r

= 5 : 2 : \(\frac{3}{2}\) ( multiply all parts by 2 )

= 10 : 4 : 3

Answer:

p : q : r​ =

10 : 4 : 3

Step-by-step explanation:

to make q equal in the two equations :

p : q = 5 : 2   x4

p : q = 20 : 8

q : r = 4 : 3    x2

q : r = 8 : 6

    So    p : q : r​ =

           20 : 8 :6

           10 : 4 : 3

find the length (in cm) of an arc of a circle with radius 6 cm if the arc subtends a central angle of 60

Answers

The length (in cm) of an arc of a circle

60°=π/3 radians

A unit circle's arc length is expressed in radians.

We may get the length of an arc using the following formula given an angle and a circle's diameter: ArcLength = (angle / 360) * (2 * pi * radius) Pi equals 22/7, diameter equals 2 * radius, and angle is in degrees.

If a unit circle's arc length is π/3,

(that is a circle with a radius of 1)

For a circle with a radius of 15, the arc length for the same angle would be

X15xπ/3

=

Know more about arc of a circle

https://brainly.com/question/17353325

#SPJ4

how many ways can rudy choose 5 pizza toppings from a menu of 20 toppings if each topping can only be chosen once?

Answers

20x19x18x17x16
Each time she can choose one less type of toppings.

which is a parametric equation for the curve 49=(x−2)2+(y+10)2?

Answers

A parametric equation for the curve 49=(x−2)2+(y+10)2 can be obtained by using the standard parameterization of a circle.

Let's first rearrange the given equation as follows:
(x-2)^2 + (y+10)² = 49

Dividing both sides by 49, we get:
[(x-2)²/49] + [(y+10)²/49] = 1

This suggests that the given equation represents an ellipse centered at (2,-10) with major and minor axes of length 2√(49) = 14 and 2√t(49) = 14, respectively.

To obtain a parametric equation for this ellipse, we can use the following parameterization:
x = 2 + 14*cos(t)
y = -10 + 14*sin(t)

Here, t is the parameter that ranges from 0 to 2*pi, and (x,y) gives the coordinates of points on the ellipse as t varies.

Note that this parametric equation satisfies the given equation for any value of t:
[(2+14*cos(t)-2)²/49] + [( -10+14*sin(t)+10)²/49] = 1

Know more about the parametric equation here:

https://brainly.com/question/30451972

#SPJ11

The number of main effects that need to be examined is _________ the number of independent variables.

Answers

Answer: the same as,

Step-by-step explanation:

Main effects is a term used to describe the impact an independent variable has on a dependent variable, this is usually averaged across any other independent variable.

Independent variables are variables that do not depend on other variables, they are variables that stand on their own.

Consider the table representing a rational function. Which equation could represent a vertical asymptote of the graph of the function?

Answers

Rounding to the nearest 10th, the surface area of the cylinder is approximately 219.8 square millimeters.

How To find the surface area of a cylinder?

To find the surface area of a cylinder, you need to add the area of the top and bottom circles to the area of the curved side.

The formula for the surface area of a cylinder is:

SA = 2πr² + 2πrh

where r is the radius of the circular base, h is the height of the cylinder, and π (pi) is approximately equal to 3.14.

First, we need to find the radius of the circular base. The length of the cylinder is given as 10mm, which is twice the radius (since the diameter is equal to the length). So, the radius is:

r = length/2 = 10mm/2 = 5mm

The height of the cylinder is given as 2mm. Now we can substitute the values into the formula:

SA = 2πr² + 2πrh

SA = 2 x 3.14 x 5² + 2 x 3.14 x 5 x 2

SA = 157 + 62.8

SA = 219.8

Rounding to the nearest 10th, the surface area of the cylinder is approximately 219.8 square millimeters.

Learn more about surface area

brainly.com/question/16112029

#SPJ11

Which statement about the function is true? the function is positive for all real values of x where x < –1. the function is negative for all real values of x where x < –3 and where x > 1. the function is positive for all real values of x where x > 0. the function is negative for all real values of x where x < –3 or x > –1.

Answers

The statement which is true about the function is that the function is negative for all real values of x where x < –3 or x > –1.

What is a Function?

A function assigns the value of each element of one set to the other specific element of another set.

The graph of the function is,

\(f(x) = -(x +3)(x - 1)\)

It is positive, its value is above x-axis. For the function above, the solutions are,

\(x+3=0\\x=-3\\x-1=0\\x=1\)

The value of x is lies between,

\(-3 < x < 1\)

Thus, the statement which is true about the function is that the function is negative for all real values of x where x < –3 or x > –1.

Learn more about Function here:

https://brainly.com/question/5245372

pls help me with this

pls help me with this

Answers

Answer:

I would but its confusing

0.45 written as a common fraction, in its simplest form, is​

Answers

Answer:

Come on man you are in high school and you can't answer this?

Step-by-step explanation:

To write 0.45 as a common fraction, we can use the fact that the decimal point separates the whole number part from the fractional part. So we have:

0.45 = 45/100

To simplify this fraction, we can divide both the numerator and the denominator by the greatest common factor, which is 5:

45/100 = (45 ÷ 5) / (100 ÷ 5) = 9/20

Therefore, 0.45 written as a common fraction, in its simplest form, is 9/20.

1. Find the component form and magnitude of the vector AB with
the following initial and terminal points.
i. A(12, 41), B(52, 33)
ii. A(8, 14), B(12, 3)
iii. A(9, -2, 5), B(8, 5, 11) (3D space)

Answers

i. To find the component form of the vector AB, we subtract the coordinates of A from the coordinates of B:
AB = <52 - 12, 33 - 41> = <40, -8>
To find the magnitude of the vector AB, we use the formula:
|AB| = sqrt((40)^2 + (-8)^2) = sqrt(1600 + 64) = sqrt(1664) ≈ 40.79

ii. Similarly, we find the component form of AB:
AB = <12 - 8, 3 - 14> = <4, -11>
And the magnitude of AB:
|AB| = sqrt((4)^2 + (-11)^2) = sqrt(157) ≈ 12.53

iii. To find the component form of AB in 3D space, we subtract the coordinates of A from the coordinates of B:
AB = <8 - 9, 5 - (-2), 11 - 5> = <-1, 7, 6>
To find the magnitude of AB, we use the formula:
|AB| = sqrt((-1)^2 + (7)^2 + (6)^2) = sqrt(86) ≈ 9.27

To learn more about Component Form & Vector : https://brainly.com/question/4088563

#SPJ11

Word problem One of the citizens has 97 silver coins. How many bronze coins would it take to equal this amount

Answers

Given: Given that a citizen have 97 silver coins.

To find : Here we need to find that how many bronze coins would it take to equal this amount.

Solution: We know, 1 silver coin=10 bronze coin

So, 97 silver coin=10×97 bronze coin

=970 bronze coin

Therefore, 970 bronze coins would it take to equal this amount.

What is the solution to this equation -8x - 40 = 8

Answers

The answer is -6 :)

1. Add 40 to both sides of the equation.

-8x-40=8

-8x-40+40=8+40

2. Add the numbers.

-8x=48

3. Divide both sides of the equation by the same term.

-8x=48

\(\frac{-8x}{-8}\)=\(\frac{48}{-8}\)

4. Cancel terms that are in both the numerator and denominator and divide the numbers.

x=-6

hope this helped! :)

Which represents the restrictions on the radical inequality \(\sqrt[4]{2x+1} \ \textgreater \ 3\)?
A. \(2x+1\geq 0\)
B. \(\sqrt{2x+1} \ \textgreater \ 0\)
C. \(2x+1\leq 0\)
D. There are no restrictions.

Answers

there are no restrictions other than 2 x + 1 > 81 on the radical inequality.

We are given the radical inequality as:

⁴√ 2 x + 1 > 3

We need to find the restriction on this inequality.

For this we will first put x on one side and other constant on the other side.

So, we get that:

2 x + 1 > 3⁴

2 x + 1 > 81

So, the restriction is not from the given options.

But, the value of 2 x + 1 should be greater than 81 for the inequality to be true.

Therefore, we get that, there are no restrictions other than 2 x + 1 > 81 on the radical inequality.

Learn more about inequality here:

https://brainly.com/question/25275758

#SPJ1

Geometry- remember what we know about verticale angles and solve for X. Then tell me the measure of angle A.

Geometry- remember what we know about verticale angles and solve for X. Then tell me the measure of angle

Answers

Answer:

m<A = 32°

Step-by-step explanation:

When angles are vertical, it means they are equal to each other:

m<A = m<C

4x - 8 = 2x + 12

4x - 2x = 12 + 8

2x = 20

x = 20/2

x = 10

Solve for A:

4x - 8

4(10) - 8

40 - 8

32

A company sells bicycle locks with a combination made of 3 numbers. Each number can be any digit from 0 to 9 and the digits may be repeated.
One of these locks is chosen at random. What is the probability that the middle digit is 0?

Answers

Answer:

1/10

Step-by-step explanation:

If it can be 10 digits (0,1,2,3,4,5,6,7,8,9) in the tens place then the probability is 0.1 or 10% to get 0

Which of the following whole numbers are perfect squares? (Please select ALL correct answers for full credit).

Which of the following whole numbers are perfect squares? (Please select ALL correct answers for full

Answers

Answer: 9, 144, 225, 10,000

Step-by-step explanation:

9=3x3

144=12x12

225=15x15

10,000=100x100

Ashley had 4/ 5 of a spool of yarn. She used 2/5 of it for her project. What fraction of the spool was used for her project? Write your answer in simplest form

Answers

Ashley used 8/25 of the spool for her project.

To determine the fraction of the spool that Ashley used for her project, we need to multiply the fraction of the spool she had (4/5) by the fraction she used (2/5):

(4/5) * (2/5) = 8/25

Know more about fraction here:

https://brainly.com/question/10354322

#SPJ11

Halliday physics obtain Heat loss rate of douple-pane window which thikness of glass layers 3mm and thikness of air layer 3mm and outside temperture -20F and inner temperure is +72F calculate it vs (w/m^2) glass Air k₁=0.78 k₂=0.026 +72F (M.K) (x) -2 OF I کمپاس 3

Answers

The heat loss rate (Q) in units of watts per square meter (W/\(m^2\)) for the given double-pane window is approximately 35.91 * A W/\(m^2\), where A is the area of the window in square meters.

Given:

Thickness of glass layers = 3mm

Thickness of air layer = 3mm

Outside temperature = -20°F = -28.9°C

Inside temperature = +72°F = 22.2°C

To calculate the heat loss rate, we will use the formula for heat conduction:

Q = (k * A * ΔT) / d

Where:

Q is the heat loss rate

k is the thermal conductivity

A is the area of the window

ΔT is the temperature difference between the inside and outside

d is the total thickness of the window

We need to convert the thicknesses of the glass and air layers to meters:

Thickness of glass layers = 3mm = 0.003m

Thickness of air layer = 3mm = 0.003m

We can now calculate the total thickness (d) of the window:

d = 2 * thickness of glass layers + thickness of air layer

d = 2 * 0.003m + 0.003m

d = 0.009m

Assuming the dimensions of the window are provided, let's consider:

Length = L meters

Width = W meters

The area of the window (A) is given by:

A = Length * Width

The thermal conductivity values in SI units are approximately:

k_glass ≈ 0.8 W/(m·K)

k_air ≈ 0.024 W/(m·K)

Now, let's calculate the temperature difference (ΔT) in Celsius:

ΔT = 22.2°C - (-28.9°C)

ΔT = 51.1°C

Substituting the values into the heat conduction formula:

Q = (k * A * ΔT) / d

Q = (0.8 W/(m·K) * A * 51.1°C) / 0.009 m

Simplifying further, we have:

Q = 44.89 * (0.8 * A) W

Therefore, the heat loss rate (Q) in units of watts per square meter (W/\(m^2\)) for the given double-pane window is approximately 35.91 * A W/\(m^2\), where A is the area of the window in square meters.

To know more about heat loss, refer here:

https://brainly.com/question/6850851

#SPJ4

Other Questions
Emanuel can throw a baseball 22 miles per hour. He throws the ball straight up in the air, releasing it at 5.6 feetabove the groundThe equation that represents the approximate height of the ball during the seconds after it is thrown ish - 16t? + 32.3t +5.6.If h represents height in feet, and t represents time in seconds, approximately how high is the ball 1.5 seconds afterit is thrown? what 5 + 5- just asnwer pls True or False: Small pox and measles were two diseases that affected workers on the Panama Canal. 15 the nurse is providing education for a client who has been prescribed phenytoin. what adverse reactions should the nurse discuss in the teaching? select all that apply. Solve for x. [tex] \frac{6}{x - 3} - \frac{12}{x + 1} = 1 [/tex]Use the digits 0-9 to enter the solution with the greatest value. the nurse is preparing to administer nasal medication via a dropper to a client with severe congestion. into which position will the nurse place the client? Hydrogen and oxygen react to produce water. Which of the following is the correctly balanced chemical equation for this reaction? a + 2 > 7Solve for a tasic b et al. (2018). shared and distinct transcriptomic cell types across neocortical areas. nature. 563 (7729): 72-78. Do all international financial transactions necessarily involve exchanging one nation's distinct currency for another? the ______________ theory contends that there are universal moral values in nature which people can discover by using the faculty of reason. please help please!!!!!!! 6.) River C is 200 miles longer than River D. If the sum of their lengths is 5,550 miles,what is the length of each river? An operations analyst counted the number of arrivals per minute at an ATM in each of 30 randomly chosen minutes. The results were: 0, 3, 3, 2, 1, 0, 1, 0, 0, 1, 1, 1, 2, 1, 0, 1, 0, 1, 2, 1, 1, 2, 1, 0, 1, 2, 0, 1, 0, 1. For the Poisson goodness-of-fit test, what is the expected frequency of the data value X In a sample of 120 children selected randomly from one town, it is found that 18 of them suffer from asthma. Which is the best choice for the null and alternative hypothesis to test that the percentage of all children in the town who suffer from asthma is different from 13% fill in the blanks with the correct present tense forms of the logical verbs. use each verb once. questions word bank reference your options are: dormir, empezar, entender, jugar, pensar, poder, preferir, querer, volver In which case did the US Supreme Court decide that prior restraint was prohibited?. Question and Answer Options on Photo Help! ZOoM in if needed :) Thank u Write and balance chemical reactions for the following reactions and name the reaction type: a. Solid sodium oxide reacts with water to form sodium hydroxide b. Solid aluminum metal reacts with aqueous zinc chloride.c. C3H8 (g) + O2 (g) --->2. Describe the three main types of radioactive decay and their properties. Do in 10 minutes Please Help! HELP ASAP! Please & Thanks Ill Thank Your Comment.